Question about exercise 3.21 of baby rudin.












0












$begingroup$


Ex 3.21 If ${E_n}$ is a sequence of closed nonempty and bounded sets in a complete metric space $X$, if $E_{n+1}subset E_n$ and if $$lim_{nrightarrowinfty} mathbb{diam} E_n=0$$
then $bigcap_{n=1}^infty E_n$ consists of exactly one point.



my proof: Let $E=bigcap_{n=1}^infty E_n$, suppose there are two points $p,q$ in $E$, then, $forall nin mathbb{N},Esubseteq E_n$ and $mathbb{diam} E_ngeq mathbb{diam} Egeq d(p,q)$, therefore there is a contradiction for
$$lim_{nrightarrowinfty} mathbb{diam} E_n=0$$
therefore, if $E$ is nonempty, $E$ has a unique point.



now show that $E$ is nonempty,
Select $x_nin E_n$, and let $$X_N:={x_n:ngeq N}$$then $$X_nin E_n,mathbb{diam} E_ngeq mathbb{diam} X_n,lim_{nrightarrowinfty} mathbb{diam} X_n=0$$ therefore sequence ${x_n}$ is cauchy sequence, and because X is complete, ${x_n}$ has limit point $x$, and $E_n$ is closed ,$$forall nin mathbb{N}, xin E_n,(because X_nsubseteq E_n)$$$$therefore xin E blacksquare$$
Question 1. Is there any error in my proof?



Question 2. why are we need boundness of $E_n$?










share|cite|improve this question









$endgroup$












  • $begingroup$
    We need boundedness so that all sets $E_n$ have a diameter at all. Plus for unbounded closed sets there are counterexamples like $E_n=[n,infty)$ in the reals.
    $endgroup$
    – Henno Brandsma
    Dec 22 '18 at 19:55












  • $begingroup$
    You mean, diameter should be define for only bounded set, isn't it?
    $endgroup$
    – 백주상
    Dec 22 '18 at 20:32


















0












$begingroup$


Ex 3.21 If ${E_n}$ is a sequence of closed nonempty and bounded sets in a complete metric space $X$, if $E_{n+1}subset E_n$ and if $$lim_{nrightarrowinfty} mathbb{diam} E_n=0$$
then $bigcap_{n=1}^infty E_n$ consists of exactly one point.



my proof: Let $E=bigcap_{n=1}^infty E_n$, suppose there are two points $p,q$ in $E$, then, $forall nin mathbb{N},Esubseteq E_n$ and $mathbb{diam} E_ngeq mathbb{diam} Egeq d(p,q)$, therefore there is a contradiction for
$$lim_{nrightarrowinfty} mathbb{diam} E_n=0$$
therefore, if $E$ is nonempty, $E$ has a unique point.



now show that $E$ is nonempty,
Select $x_nin E_n$, and let $$X_N:={x_n:ngeq N}$$then $$X_nin E_n,mathbb{diam} E_ngeq mathbb{diam} X_n,lim_{nrightarrowinfty} mathbb{diam} X_n=0$$ therefore sequence ${x_n}$ is cauchy sequence, and because X is complete, ${x_n}$ has limit point $x$, and $E_n$ is closed ,$$forall nin mathbb{N}, xin E_n,(because X_nsubseteq E_n)$$$$therefore xin E blacksquare$$
Question 1. Is there any error in my proof?



Question 2. why are we need boundness of $E_n$?










share|cite|improve this question









$endgroup$












  • $begingroup$
    We need boundedness so that all sets $E_n$ have a diameter at all. Plus for unbounded closed sets there are counterexamples like $E_n=[n,infty)$ in the reals.
    $endgroup$
    – Henno Brandsma
    Dec 22 '18 at 19:55












  • $begingroup$
    You mean, diameter should be define for only bounded set, isn't it?
    $endgroup$
    – 백주상
    Dec 22 '18 at 20:32
















0












0








0





$begingroup$


Ex 3.21 If ${E_n}$ is a sequence of closed nonempty and bounded sets in a complete metric space $X$, if $E_{n+1}subset E_n$ and if $$lim_{nrightarrowinfty} mathbb{diam} E_n=0$$
then $bigcap_{n=1}^infty E_n$ consists of exactly one point.



my proof: Let $E=bigcap_{n=1}^infty E_n$, suppose there are two points $p,q$ in $E$, then, $forall nin mathbb{N},Esubseteq E_n$ and $mathbb{diam} E_ngeq mathbb{diam} Egeq d(p,q)$, therefore there is a contradiction for
$$lim_{nrightarrowinfty} mathbb{diam} E_n=0$$
therefore, if $E$ is nonempty, $E$ has a unique point.



now show that $E$ is nonempty,
Select $x_nin E_n$, and let $$X_N:={x_n:ngeq N}$$then $$X_nin E_n,mathbb{diam} E_ngeq mathbb{diam} X_n,lim_{nrightarrowinfty} mathbb{diam} X_n=0$$ therefore sequence ${x_n}$ is cauchy sequence, and because X is complete, ${x_n}$ has limit point $x$, and $E_n$ is closed ,$$forall nin mathbb{N}, xin E_n,(because X_nsubseteq E_n)$$$$therefore xin E blacksquare$$
Question 1. Is there any error in my proof?



Question 2. why are we need boundness of $E_n$?










share|cite|improve this question









$endgroup$




Ex 3.21 If ${E_n}$ is a sequence of closed nonempty and bounded sets in a complete metric space $X$, if $E_{n+1}subset E_n$ and if $$lim_{nrightarrowinfty} mathbb{diam} E_n=0$$
then $bigcap_{n=1}^infty E_n$ consists of exactly one point.



my proof: Let $E=bigcap_{n=1}^infty E_n$, suppose there are two points $p,q$ in $E$, then, $forall nin mathbb{N},Esubseteq E_n$ and $mathbb{diam} E_ngeq mathbb{diam} Egeq d(p,q)$, therefore there is a contradiction for
$$lim_{nrightarrowinfty} mathbb{diam} E_n=0$$
therefore, if $E$ is nonempty, $E$ has a unique point.



now show that $E$ is nonempty,
Select $x_nin E_n$, and let $$X_N:={x_n:ngeq N}$$then $$X_nin E_n,mathbb{diam} E_ngeq mathbb{diam} X_n,lim_{nrightarrowinfty} mathbb{diam} X_n=0$$ therefore sequence ${x_n}$ is cauchy sequence, and because X is complete, ${x_n}$ has limit point $x$, and $E_n$ is closed ,$$forall nin mathbb{N}, xin E_n,(because X_nsubseteq E_n)$$$$therefore xin E blacksquare$$
Question 1. Is there any error in my proof?



Question 2. why are we need boundness of $E_n$?







general-topology analysis






share|cite|improve this question













share|cite|improve this question











share|cite|improve this question




share|cite|improve this question










asked Dec 22 '18 at 19:48









백주상백주상

1309




1309












  • $begingroup$
    We need boundedness so that all sets $E_n$ have a diameter at all. Plus for unbounded closed sets there are counterexamples like $E_n=[n,infty)$ in the reals.
    $endgroup$
    – Henno Brandsma
    Dec 22 '18 at 19:55












  • $begingroup$
    You mean, diameter should be define for only bounded set, isn't it?
    $endgroup$
    – 백주상
    Dec 22 '18 at 20:32




















  • $begingroup$
    We need boundedness so that all sets $E_n$ have a diameter at all. Plus for unbounded closed sets there are counterexamples like $E_n=[n,infty)$ in the reals.
    $endgroup$
    – Henno Brandsma
    Dec 22 '18 at 19:55












  • $begingroup$
    You mean, diameter should be define for only bounded set, isn't it?
    $endgroup$
    – 백주상
    Dec 22 '18 at 20:32


















$begingroup$
We need boundedness so that all sets $E_n$ have a diameter at all. Plus for unbounded closed sets there are counterexamples like $E_n=[n,infty)$ in the reals.
$endgroup$
– Henno Brandsma
Dec 22 '18 at 19:55






$begingroup$
We need boundedness so that all sets $E_n$ have a diameter at all. Plus for unbounded closed sets there are counterexamples like $E_n=[n,infty)$ in the reals.
$endgroup$
– Henno Brandsma
Dec 22 '18 at 19:55














$begingroup$
You mean, diameter should be define for only bounded set, isn't it?
$endgroup$
– 백주상
Dec 22 '18 at 20:32






$begingroup$
You mean, diameter should be define for only bounded set, isn't it?
$endgroup$
– 백주상
Dec 22 '18 at 20:32












1 Answer
1






active

oldest

votes


















2












$begingroup$

For the condition that $operatorname{diam}(E_n) to 0$ to make sense we must have that at most finitely many $E_n$ have infinite diameter, and then we could just start the sequence at the first finite diameter set. All sets in the sequence are then bounded, with diameters tending to $0$. So the condition "forces" bounded sets, and anyway, there are counterexamples for unbounded closed sets as $E_n = [n, infty)$ in the reals, another good reason to stay with bounded sets (and if one $E_n$ is bounded, the tail will consist of bounded sets by nestedness and we could just as well have used only bounded sets).



As to your proof, the idea is fine, but the write-up could sometimes be better.



Unicity is fine, if $p neq q $ are both in $cap_n E_n$, for all $n$ $operatorname{diam}(E_n) ge d(p,q) >0$ and the limit condition cannot hold.



Existence: just pick $x_n in E_n$ arbitarily. If $varepsilon >0$, pick $N$ so that $operatorname{diam}(E_N) < varepsilon$. Then for $n,m ge N$, we know that $x_n, x_m in E_{m'} subseteq E_N$ where $m'=max(n,m) ge N$ because the sets are nested and thus $$d(x_n, x_m) le operatorname{diam}(E_{m'}) le operatorname{diam}(E_N) < varepsilon$$ and $(x_n)$ is Cauchy because $varepsilon>0$ was arbitrary.



We know by completeness that $(x_n)$ converges to some $p in X$.
Fix $n$ for now. Then for all $m ge n$, $x_m in E_m subseteq E_n$ and as $E_n$ is closed and $(x_m)_{m ge n} to p$ also, $p in E_n$. As $n$ was arbitary, $p in bigcap_n E_n$ as required. Your sets $X_n$ are superfluous.






share|cite|improve this answer









$endgroup$













  • $begingroup$
    Thank you for your answer, but I have a question, I don't know how $E_n=[n,infty)$ is counterexample of our problem, if we remove boundness condition, actually I don't know how we explain $lim_{nrightarrow infty}mathbb{diam} [n,infty)=0$ in $mathbb{R}$. Can you give some helps?
    $endgroup$
    – 백주상
    Dec 25 '18 at 17:14








  • 1




    $begingroup$
    @백주상 the example is meant to illustrate that we need bounded sets: the sets are closed, nested but have empty intersection. They all have infinite diameter so they don’t obey the final condition which is thus necessary.
    $endgroup$
    – Henno Brandsma
    Dec 25 '18 at 18:43











Your Answer





StackExchange.ifUsing("editor", function () {
return StackExchange.using("mathjaxEditing", function () {
StackExchange.MarkdownEditor.creationCallbacks.add(function (editor, postfix) {
StackExchange.mathjaxEditing.prepareWmdForMathJax(editor, postfix, [["$", "$"], ["\\(","\\)"]]);
});
});
}, "mathjax-editing");

StackExchange.ready(function() {
var channelOptions = {
tags: "".split(" "),
id: "69"
};
initTagRenderer("".split(" "), "".split(" "), channelOptions);

StackExchange.using("externalEditor", function() {
// Have to fire editor after snippets, if snippets enabled
if (StackExchange.settings.snippets.snippetsEnabled) {
StackExchange.using("snippets", function() {
createEditor();
});
}
else {
createEditor();
}
});

function createEditor() {
StackExchange.prepareEditor({
heartbeatType: 'answer',
autoActivateHeartbeat: false,
convertImagesToLinks: true,
noModals: true,
showLowRepImageUploadWarning: true,
reputationToPostImages: 10,
bindNavPrevention: true,
postfix: "",
imageUploader: {
brandingHtml: "Powered by u003ca class="icon-imgur-white" href="https://imgur.com/"u003eu003c/au003e",
contentPolicyHtml: "User contributions licensed under u003ca href="https://creativecommons.org/licenses/by-sa/3.0/"u003ecc by-sa 3.0 with attribution requiredu003c/au003e u003ca href="https://stackoverflow.com/legal/content-policy"u003e(content policy)u003c/au003e",
allowUrls: true
},
noCode: true, onDemand: true,
discardSelector: ".discard-answer"
,immediatelyShowMarkdownHelp:true
});


}
});














draft saved

draft discarded


















StackExchange.ready(
function () {
StackExchange.openid.initPostLogin('.new-post-login', 'https%3a%2f%2fmath.stackexchange.com%2fquestions%2f3049785%2fquestion-about-exercise-3-21-of-baby-rudin%23new-answer', 'question_page');
}
);

Post as a guest















Required, but never shown

























1 Answer
1






active

oldest

votes








1 Answer
1






active

oldest

votes









active

oldest

votes






active

oldest

votes









2












$begingroup$

For the condition that $operatorname{diam}(E_n) to 0$ to make sense we must have that at most finitely many $E_n$ have infinite diameter, and then we could just start the sequence at the first finite diameter set. All sets in the sequence are then bounded, with diameters tending to $0$. So the condition "forces" bounded sets, and anyway, there are counterexamples for unbounded closed sets as $E_n = [n, infty)$ in the reals, another good reason to stay with bounded sets (and if one $E_n$ is bounded, the tail will consist of bounded sets by nestedness and we could just as well have used only bounded sets).



As to your proof, the idea is fine, but the write-up could sometimes be better.



Unicity is fine, if $p neq q $ are both in $cap_n E_n$, for all $n$ $operatorname{diam}(E_n) ge d(p,q) >0$ and the limit condition cannot hold.



Existence: just pick $x_n in E_n$ arbitarily. If $varepsilon >0$, pick $N$ so that $operatorname{diam}(E_N) < varepsilon$. Then for $n,m ge N$, we know that $x_n, x_m in E_{m'} subseteq E_N$ where $m'=max(n,m) ge N$ because the sets are nested and thus $$d(x_n, x_m) le operatorname{diam}(E_{m'}) le operatorname{diam}(E_N) < varepsilon$$ and $(x_n)$ is Cauchy because $varepsilon>0$ was arbitrary.



We know by completeness that $(x_n)$ converges to some $p in X$.
Fix $n$ for now. Then for all $m ge n$, $x_m in E_m subseteq E_n$ and as $E_n$ is closed and $(x_m)_{m ge n} to p$ also, $p in E_n$. As $n$ was arbitary, $p in bigcap_n E_n$ as required. Your sets $X_n$ are superfluous.






share|cite|improve this answer









$endgroup$













  • $begingroup$
    Thank you for your answer, but I have a question, I don't know how $E_n=[n,infty)$ is counterexample of our problem, if we remove boundness condition, actually I don't know how we explain $lim_{nrightarrow infty}mathbb{diam} [n,infty)=0$ in $mathbb{R}$. Can you give some helps?
    $endgroup$
    – 백주상
    Dec 25 '18 at 17:14








  • 1




    $begingroup$
    @백주상 the example is meant to illustrate that we need bounded sets: the sets are closed, nested but have empty intersection. They all have infinite diameter so they don’t obey the final condition which is thus necessary.
    $endgroup$
    – Henno Brandsma
    Dec 25 '18 at 18:43
















2












$begingroup$

For the condition that $operatorname{diam}(E_n) to 0$ to make sense we must have that at most finitely many $E_n$ have infinite diameter, and then we could just start the sequence at the first finite diameter set. All sets in the sequence are then bounded, with diameters tending to $0$. So the condition "forces" bounded sets, and anyway, there are counterexamples for unbounded closed sets as $E_n = [n, infty)$ in the reals, another good reason to stay with bounded sets (and if one $E_n$ is bounded, the tail will consist of bounded sets by nestedness and we could just as well have used only bounded sets).



As to your proof, the idea is fine, but the write-up could sometimes be better.



Unicity is fine, if $p neq q $ are both in $cap_n E_n$, for all $n$ $operatorname{diam}(E_n) ge d(p,q) >0$ and the limit condition cannot hold.



Existence: just pick $x_n in E_n$ arbitarily. If $varepsilon >0$, pick $N$ so that $operatorname{diam}(E_N) < varepsilon$. Then for $n,m ge N$, we know that $x_n, x_m in E_{m'} subseteq E_N$ where $m'=max(n,m) ge N$ because the sets are nested and thus $$d(x_n, x_m) le operatorname{diam}(E_{m'}) le operatorname{diam}(E_N) < varepsilon$$ and $(x_n)$ is Cauchy because $varepsilon>0$ was arbitrary.



We know by completeness that $(x_n)$ converges to some $p in X$.
Fix $n$ for now. Then for all $m ge n$, $x_m in E_m subseteq E_n$ and as $E_n$ is closed and $(x_m)_{m ge n} to p$ also, $p in E_n$. As $n$ was arbitary, $p in bigcap_n E_n$ as required. Your sets $X_n$ are superfluous.






share|cite|improve this answer









$endgroup$













  • $begingroup$
    Thank you for your answer, but I have a question, I don't know how $E_n=[n,infty)$ is counterexample of our problem, if we remove boundness condition, actually I don't know how we explain $lim_{nrightarrow infty}mathbb{diam} [n,infty)=0$ in $mathbb{R}$. Can you give some helps?
    $endgroup$
    – 백주상
    Dec 25 '18 at 17:14








  • 1




    $begingroup$
    @백주상 the example is meant to illustrate that we need bounded sets: the sets are closed, nested but have empty intersection. They all have infinite diameter so they don’t obey the final condition which is thus necessary.
    $endgroup$
    – Henno Brandsma
    Dec 25 '18 at 18:43














2












2








2





$begingroup$

For the condition that $operatorname{diam}(E_n) to 0$ to make sense we must have that at most finitely many $E_n$ have infinite diameter, and then we could just start the sequence at the first finite diameter set. All sets in the sequence are then bounded, with diameters tending to $0$. So the condition "forces" bounded sets, and anyway, there are counterexamples for unbounded closed sets as $E_n = [n, infty)$ in the reals, another good reason to stay with bounded sets (and if one $E_n$ is bounded, the tail will consist of bounded sets by nestedness and we could just as well have used only bounded sets).



As to your proof, the idea is fine, but the write-up could sometimes be better.



Unicity is fine, if $p neq q $ are both in $cap_n E_n$, for all $n$ $operatorname{diam}(E_n) ge d(p,q) >0$ and the limit condition cannot hold.



Existence: just pick $x_n in E_n$ arbitarily. If $varepsilon >0$, pick $N$ so that $operatorname{diam}(E_N) < varepsilon$. Then for $n,m ge N$, we know that $x_n, x_m in E_{m'} subseteq E_N$ where $m'=max(n,m) ge N$ because the sets are nested and thus $$d(x_n, x_m) le operatorname{diam}(E_{m'}) le operatorname{diam}(E_N) < varepsilon$$ and $(x_n)$ is Cauchy because $varepsilon>0$ was arbitrary.



We know by completeness that $(x_n)$ converges to some $p in X$.
Fix $n$ for now. Then for all $m ge n$, $x_m in E_m subseteq E_n$ and as $E_n$ is closed and $(x_m)_{m ge n} to p$ also, $p in E_n$. As $n$ was arbitary, $p in bigcap_n E_n$ as required. Your sets $X_n$ are superfluous.






share|cite|improve this answer









$endgroup$



For the condition that $operatorname{diam}(E_n) to 0$ to make sense we must have that at most finitely many $E_n$ have infinite diameter, and then we could just start the sequence at the first finite diameter set. All sets in the sequence are then bounded, with diameters tending to $0$. So the condition "forces" bounded sets, and anyway, there are counterexamples for unbounded closed sets as $E_n = [n, infty)$ in the reals, another good reason to stay with bounded sets (and if one $E_n$ is bounded, the tail will consist of bounded sets by nestedness and we could just as well have used only bounded sets).



As to your proof, the idea is fine, but the write-up could sometimes be better.



Unicity is fine, if $p neq q $ are both in $cap_n E_n$, for all $n$ $operatorname{diam}(E_n) ge d(p,q) >0$ and the limit condition cannot hold.



Existence: just pick $x_n in E_n$ arbitarily. If $varepsilon >0$, pick $N$ so that $operatorname{diam}(E_N) < varepsilon$. Then for $n,m ge N$, we know that $x_n, x_m in E_{m'} subseteq E_N$ where $m'=max(n,m) ge N$ because the sets are nested and thus $$d(x_n, x_m) le operatorname{diam}(E_{m'}) le operatorname{diam}(E_N) < varepsilon$$ and $(x_n)$ is Cauchy because $varepsilon>0$ was arbitrary.



We know by completeness that $(x_n)$ converges to some $p in X$.
Fix $n$ for now. Then for all $m ge n$, $x_m in E_m subseteq E_n$ and as $E_n$ is closed and $(x_m)_{m ge n} to p$ also, $p in E_n$. As $n$ was arbitary, $p in bigcap_n E_n$ as required. Your sets $X_n$ are superfluous.







share|cite|improve this answer












share|cite|improve this answer



share|cite|improve this answer










answered Dec 22 '18 at 22:29









Henno BrandsmaHenno Brandsma

111k348120




111k348120












  • $begingroup$
    Thank you for your answer, but I have a question, I don't know how $E_n=[n,infty)$ is counterexample of our problem, if we remove boundness condition, actually I don't know how we explain $lim_{nrightarrow infty}mathbb{diam} [n,infty)=0$ in $mathbb{R}$. Can you give some helps?
    $endgroup$
    – 백주상
    Dec 25 '18 at 17:14








  • 1




    $begingroup$
    @백주상 the example is meant to illustrate that we need bounded sets: the sets are closed, nested but have empty intersection. They all have infinite diameter so they don’t obey the final condition which is thus necessary.
    $endgroup$
    – Henno Brandsma
    Dec 25 '18 at 18:43


















  • $begingroup$
    Thank you for your answer, but I have a question, I don't know how $E_n=[n,infty)$ is counterexample of our problem, if we remove boundness condition, actually I don't know how we explain $lim_{nrightarrow infty}mathbb{diam} [n,infty)=0$ in $mathbb{R}$. Can you give some helps?
    $endgroup$
    – 백주상
    Dec 25 '18 at 17:14








  • 1




    $begingroup$
    @백주상 the example is meant to illustrate that we need bounded sets: the sets are closed, nested but have empty intersection. They all have infinite diameter so they don’t obey the final condition which is thus necessary.
    $endgroup$
    – Henno Brandsma
    Dec 25 '18 at 18:43
















$begingroup$
Thank you for your answer, but I have a question, I don't know how $E_n=[n,infty)$ is counterexample of our problem, if we remove boundness condition, actually I don't know how we explain $lim_{nrightarrow infty}mathbb{diam} [n,infty)=0$ in $mathbb{R}$. Can you give some helps?
$endgroup$
– 백주상
Dec 25 '18 at 17:14






$begingroup$
Thank you for your answer, but I have a question, I don't know how $E_n=[n,infty)$ is counterexample of our problem, if we remove boundness condition, actually I don't know how we explain $lim_{nrightarrow infty}mathbb{diam} [n,infty)=0$ in $mathbb{R}$. Can you give some helps?
$endgroup$
– 백주상
Dec 25 '18 at 17:14






1




1




$begingroup$
@백주상 the example is meant to illustrate that we need bounded sets: the sets are closed, nested but have empty intersection. They all have infinite diameter so they don’t obey the final condition which is thus necessary.
$endgroup$
– Henno Brandsma
Dec 25 '18 at 18:43




$begingroup$
@백주상 the example is meant to illustrate that we need bounded sets: the sets are closed, nested but have empty intersection. They all have infinite diameter so they don’t obey the final condition which is thus necessary.
$endgroup$
– Henno Brandsma
Dec 25 '18 at 18:43


















draft saved

draft discarded




















































Thanks for contributing an answer to Mathematics Stack Exchange!


  • Please be sure to answer the question. Provide details and share your research!

But avoid



  • Asking for help, clarification, or responding to other answers.

  • Making statements based on opinion; back them up with references or personal experience.


Use MathJax to format equations. MathJax reference.


To learn more, see our tips on writing great answers.




draft saved


draft discarded














StackExchange.ready(
function () {
StackExchange.openid.initPostLogin('.new-post-login', 'https%3a%2f%2fmath.stackexchange.com%2fquestions%2f3049785%2fquestion-about-exercise-3-21-of-baby-rudin%23new-answer', 'question_page');
}
);

Post as a guest















Required, but never shown





















































Required, but never shown














Required, but never shown












Required, but never shown







Required, but never shown

































Required, but never shown














Required, but never shown












Required, but never shown







Required, but never shown







Popular posts from this blog

To store a contact into the json file from server.js file using a class in NodeJS

Redirect URL with Chrome Remote Debugging Android Devices

Dieringhausen